LSAT and Law School Admissions Forum

Get expert LSAT preparation and law school admissions advice from PowerScore Test Preparation.

 Administrator
PowerScore Staff
  • PowerScore Staff
  • Posts: 8917
  • Joined: Feb 02, 2011
|
#45692
Complete Question Explanation
(The complete setup for this game can be found here: lsat/viewtopic.php?t=16967)

The correct answer choice is (A)

From the inferences we know that W cannot be selected with F (W requires H, and H cannot be with F), so answer choice (A) is correct.

Get the most out of your LSAT Prep Plus subscription.

Analyze and track your performance with our Testing and Analytics Package.